zoukankan      html  css  js  c++  java
  • 2015年北京大学自主招生选拔录取考试数学部分

    一、选择题(选对得10分,不选得0分,选错扣5分)

    1、整数$x,y,z$满足$xy+yz+zx=1$,则$(1+x^2)(1+y^2)(1+z^2)$可能取到的值为(        )
    A.$16900$
    B.$17900$
    C.$18900$
    D.前三个答案都不对
    2、在不超过$99$的正整数中选出$50$个不同的正整数,已知这$50$个数中任两个的和都不等于$99$,也不等于$100$.这$50$个数的和可能等于(        )
    A.$3524$
    B.$3624$
    C.$3724$
    D.前三个答案都不对
    3、已知$xinleft[0,dfrac{pi}2 ight]$,对任意实数$a$,函数$y=cos^2x-2acos x+1$的最小值记为$g(a)$,则当$a$取遍所有实数时,$g(a)$的最大值为(        )
    A.$1$
    B.$2$
    C.$3$
    D.前三个答案都不对
    4、已知$10^{20}-2^{20}$是$2^n$的整数倍,则正整数$n$的最大值为(        )
    A.$21$
    B.$22$
    C.$23$
    D.前三个答案都不对
    5、在凸四边形$ABCD$中,$BC=4$,$angle ADC=60^circ$,$angle BAD=90^circ$,四边形$ABCD$的面积等于$dfrac{ABcdot CD+BCcdot AD}{2}$,则$CD$的长(精确到小数点后$1$位)为(        )
    A.$6.9$
    B.$7.1$
    C.$7.3$
    D.前三个答案都不对
    二、填空题(填空题共5小题;请把每小题的正确答案填在横线上,每题10分)
    6、满足等式$left(1+dfrac 1x ight)^{x+1}=left(1+dfrac{1}{2015} ight)^{2015}$的整数$x$的个数是_______.
    7、已知$a,b,c,din[2,4]$,则$dfrac{(ab+cd)^2}{(a^2+d^2)(b^2+c^2)}$的最大值与最小值的和为_______.
    8、已知对于任意的实数$xin [1,5]$,$left|x^2+px+q ight|leqslant 2$,不超过$sqrt{p^2+q^2}$的最大整数是_______.
    9、设$x=dfrac{b^2+c^2-a^2}{2bc}$,$y=dfrac{c^2+a^2-b^2}{2ca}$,$z=dfrac{a^2+b^2-c^2}{2ab}$,且$x+y+z=1$,则$x^{2015}+y^{2015}+z^{2015}$的值为_______.
    10、设$A_1,A_2,cdots ,A_n$都是$9$元集合${1,2,cdots ,9}$的子集,已知$|A_i|$为奇数,$1leqslant ileqslant n$,$left|A_icap A_j ight|$为偶数,$1leqslant i eq jleqslant n$,则$n$的最大值为_______.
    参考答案
    一、选择题
    1、A
    $(1+x^2)(1+y^2)(1+z^2)=left( (x+y)(y+z)(z+x) ight)^2$.令$$egin{cases}x+y=2,\y+z=5,\z+x=13,end{cases}$$解得$$egin{cases}x=5,\y=-3,\z=8.end{cases}$$经检验,这组解满足题意,此时$(1+x^2)(1+y^2)(1+z^2)=16900$.
    2、D
    考虑将$1,2,cdots,99$这$99$个正整数分成如下$50$组:[(1,99),(2,98),cdots,(47,53),(48,52),(49,51),(50).]若选出的$50$个不同的正整数中没有$50$,则必有$2$个数位于[(1,99),(2,98),cdots,(47,53),(48,52),(49,51)]中的同一组,不合题意.所以这$50$个不同的正整数中必有$50$,而[(1,99),(2,98),cdots,(47,53),(48,52),(49,51)]中,每组有且只有一个数被选中.
    因为$50+49=99$,所以$(49,51)$中选$51$;因为$51+48=99$,所以$(48,52)$中选$52$;以此类推,可得$50,51,52,cdots,98,99$是唯一可能的选法.
    经检验,选$50,51,52,cdots,98,99$满足题意,此时$50+51+cdots+98+99=3725$,故选D.
    3、A
    令$t=cos xin [0,1]$,令$h(t)=t^2-2at+1,tin [0,1]$,则[g(a)=egin{cases}1,&(a<0)\1-a^2,&(0leqslant a leqslant 1)\2-2a,&(a>1)end{cases} ]故$g(a)$的最大值为$1$($aleqslant 0$时等号成立).
    4、D
    $10^{20}-2^{20}=2^{20} left(5^{20}-1 ight)=2^{20} left(5^{10}+1 ight)left(5^{5}+1 ight)left(5-1 ight)left(5^4+5^3+5^2+5+1 ight)$,而$5^{10}+1$模$4$余$2$,$5^{5}+1$模$4$余$2$,$5^4+5^3+5^2+5+1$为奇数,故正整数$n$的最大值为$24$.
    5、A
    设四边形$ABCD$的面积为$S$,直线$AC,BD$的夹角为$ heta$,则[S=dfrac{ACcdot BDcdot sin heta}{2}leqslantdfrac{ABcdot CD+BCcdot AD}{2}cdotsin hetaleqslant dfrac{ABcdot CD+BCcdot AD}{2}, ] 由题意,$S=dfrac{ABcdot CD+BCcdot AD}{2}$,所以$A,B,C,D$四点共圆,且$ACperp BD$.
    故$CD=4sqrt{3}approx 6.9 $,选A.
    二、填空题
    6、$1$
    若$x$为正整数,则$$left(1+dfrac{1}{x} ight)^{x+1}>mathrm e>left(1+dfrac{1}{2015} ight)^{2015} ,$$
    若$x$为负整数,令$x=-n(nin mathbf N_+,ngeqslant 2)$,则$$left(1+dfrac{1}{x} ight)^{x+1}=left(1+dfrac{1}{n-1} ight)^{n-1}.$$
    因为数列$left(1+dfrac{1}{n-1} ight)^{n-1}(nin mathbf N_+,ngeqslant 2)$关于$n$单调递增,故当且仅当$x=-2016$时,有$$left(1+dfrac 1x ight)^{x+1}=left(1+dfrac{1}{2015} ight)^{2015}.$$
    7、$dfrac{41}{25}$.
    注意到$$(a^2+d^2)(b^2+c^2)=(ab+cd)^2+(ac-bd)^2,$$于是[egin{split} dfrac{(ab+cd)^2}{(a^2+d^2)(b^2+c^2)}&=dfrac{(ab+cd)^2}{(ab+cd)^2+(ac-bd)^2}\ &=dfrac{1}{1+left(dfrac{ac-bd}{ab+cd} ight)^2},end{split} ]显然当$ac-bd=0$时,原式取得最大值为$1$.
    接下来考虑$left|dfrac{ac-bd}{ab+cd} ight|$的最大值.
    由于$$left|dfrac{ac-bd}{ab+cd} ight|=left|dfrac{frac ad-frac bc}{frac adcdot frac bc+1} ight|,$$令$dfrac ad= an alpha$,$dfrac bc= an eta$,则问题等价于当$alpha,etain left[arctan dfrac 12,arctan 2 ight]$时,求$ an|alpha-eta|$的最大值,显然为$$ anleft(arctan 2-arctan dfrac 12 ight)=dfrac 34.$$
    因此原式的最小值为$dfrac{16}{25}$.
    注    可以看做向量$(a,d)$和$(b,c)$夹角余弦的平方.
    8、$9$
    注意到$y=x^2+px+q$,$xin [1,5]$满足$-2leqslant yleqslant 2$,因此符合题意的二次函数只有两个:$$y=x^2-6x+7,y=-x^2+6x-7.$$
    9、$1$
    由$x+y+z=1$,可得[egin{split} &ab^2+ac^2-a^3+bc^2+a^2b-b^3+a^2c+b^2c-c^3-2abc\=&left(ab^2+a^2b-a^3-b^3 ight )+left(ac^2+bc^2-c^3 ight )+left(a^2c+b^2c-2abc ight)\=&-(a+b)(a-b)^2+c^2(a+b-c)+c(a-b)^2\=&-(a-b-c)(b-c-a)(c-a-b)\=&0,end{split} ]所以$a=b+c$或$b=c+a$或$c=a+b$,故$x^{2015}+y^{2015}+z^{2015}=1$.
    10、 $9$
    构造是容易的,取$A_i={i}$,$i=1,2,cdots ,9$即可.
    用$0,1$表示集合中的元素是否在子集中,如$A_1={1,3,4,5,9}$,则记$$A_1=(1,0,1,1,1,0,0,0,1),$$那么$$A_icdot A_j=left|A_icap A_j ight|.$$
    显然,如果当$ngeqslant 10$时,必然存在$m$个向量线性相关,不妨设$$lambda_1A_1+lambda_2A_2+cdots +lambda_mA_m=(0,0,cdots ,0),$$其中$lambda_iinmathcal Z$($i=1,2,cdots ,m$),$lambda_1=1$.
    此时考虑$$A_1cdotleft(lambda_1A_1+lambda_2A_2+cdots +lambda_mA_m ight),$$那么根据题意有$A_1cdot A_1$为奇数,而$A_1cdot A_i$($i=2,3,cdots ,m$)为偶数,这样就推出了矛盾.
    因此所求$n$的最大值为$9$.
    注    用这个方法,可以得出$n$元集合至多有$n$个包含奇数个元素的子集,使得这些子集中任意两个的交集均包含偶数个元素.
     
    2014年北约自主招生试题
    lanqi.org 2014年6月1
    第1页
     
    一、填空题
    1、有一个圆心角是(60^circ) ,面积是(6pi)的扇形围成一个圆锥,则圆锥的表面积是_______.
    2、已知(f(x))满足(fleft(dfrac {a+2b}{3} ight)=dfrac {f(a)+2f(b)}{3}),(f(1)=1),(f(4)=7),则(f(2014)=)_______.
    3、(10)个人分成(3)人、(3)人、(4)人三组,共有_______种不同的分组方法.
    4、已知(f (x)=lg{left(x^2-2ax+a ight)})的值域为(mathcal R) ,则实数(a)的取值范围为_______.
    5、已知(x<0),(y < 0),(x+y=-1),则(xy+dfrac 1{xy})的最_______值是_______.
    6、(f(x)=arctan{dfrac {2+2x}{1-4x}}+C)在(left(-dfrac 14,dfrac 14 ight))上为奇函数,则(C)的值是_______.
    二、解答题
    7、证明:( an {3^circ})是无理数.
    8、已知 (y=f(x)),(y=g(x))都是二次函数,方程(3f(x)+g(x)=0)和方程(f(x)-g(x)=0)都只有一个重根,方程(f(x)=0)有两个不等实根.证明:方程(g(x)=0)没有实数根.
    9、已知数列(left{a_n ight})是(13)项的等差数列,集合[A=left{a_i+a_j+a_kleft| ight.1leqslant i < j < k leqslant 13,i,j,kin f N^* ight},]则(0,dfrac 72,dfrac {16}3)能否同时在集合(A)中?
    10、已知({x_1}{x_2} cdots {x_n} = 1),({x_i} > 0),(i = 1,2, cdots ,n),求证:[left( {sqrt 2 + {x_1}} ight)left( {sqrt 2 + {x_2}} ight) cdots left( {sqrt 2 + {x_n}} ight) geqslant {left( {sqrt 2 + 1} ight)^n}.]
    参考答案
    一、填空题
    1、(7{ m pi})
    2、(4027)
    3、(2100)
    4、(left( { - infty , 0} ight] cup left[ {1 , + infty } ight))
    5、小,(dfrac{{17}}{4})
    6、( - arctan 2)
    二、解答题
    7、用反证法. 假设( an 3^circ )是有理数,则( an left( {k cdot 3^circ } ight))均为有理数,于是( an 30^circ )为有理数,矛盾. 因此( an 3^circ )是无理数.
    8、设函数(Aleft( x ight) = 3fleft( x ight) + gleft( x ight)),(Bleft( x ight) = fleft( x ight) - gleft( x ight)),则函数(Aleft( x ight))、(Bleft( x ight))均为二次函数,此时[4fleft( x ight) = Aleft( x ight) + Bleft( x ight).] 考虑到方程(fleft( x ight) = 0)有两个不等实根,于是方程(Aleft( x ight) + Bleft( x ight) = 0)有两个不等实根. 因此抛物线(y = Aleft( x ight)),(y = Bleft( x ight))的开口方向必然不同,且零点亦不相同. 于是[gleft( x ight) = dfrac{{Aleft( x ight) - 3Bleft( x ight)}}{4}]必然恒大于(0)或恒小于(0). 因此原命题得证.
    9、容易证明:将集合(A)中的所有数从小到大排列,则可以得到一个与数列(left{ {{a_n}} ight})的公差相同的等差数列(left{ {{b_n}} ight}). 设数列(left{ {{a_n}} ight})的公差为(d),不妨设(d geqslant 0). (i)若(d = 0),则显然不符合题意; (ii)若(d > 0),则数列(left{ {{b_n}} ight})中的最小项为({a_1} + {a_2} + {a_3}),最大项为({a_{11}} + {a_{12}} + {a_{13}}).其中至多有[left( {11 + 12 + 13} ight) - left( {1 + 2 + 3} ight) + 1 = 31]项,且任意两项的差均为公差(d)的整数倍. 不失一般性,将(0 , dfrac{7}{2} , dfrac{{16}}{3})看作(0 , 21 , 32)(所有的数同时扩大(6)倍即可).由于(left( {21 , 32} ight)= 1),而(d|1),于是(d leqslant 1),因此同时包含(0 , 21 , 32)的等差数列至少有(32)项. 这与数列(left{ {{b_n}} ight})中至多有(31)项矛盾,因此(0 , dfrac{7}{2} , dfrac{{16}}{3})不能同时在集合(A)中.
    10、法一
    直接展开,对应项用均值不等式即可.
    法二 当(n = 1)时,不等式显然成立. 假设命题对不超过(n)的正整数均成立,则命题对(n + 1): 不妨设({x_1} geqslant 1),({x_2} leqslant 1),而[left( {{x_1}{x_2}} ight){x_3} cdots {x_n}{x_{n + 1}} = 1,]于是[left( {sqrt 2 + {x_1}{x_2}} ight)left( {sqrt 2 + {x_3}} ight) cdots left( {sqrt 2 + {x_{n + 1}}} ight) geqslant {left( {sqrt 2 + 1} ight)^n}.] 因此只需要证明[dfrac{{left( {sqrt 2 + {x_1}} ight)left( {sqrt 2 + {x_2}} ight)}}{{sqrt 2 + {x_1}{x_2}}} geqslant sqrt 2 + 1.] 用分析法: 该不等式成立[egin{split} & Leftarrow left( {sqrt 2 + {x_1}} ight)left( {sqrt 2 + {x_2}} ight) geqslant left( {sqrt 2 + {x_1}{x_2}} ight)left( {sqrt 2 + 1} ight)\ & Leftarrow 2 + sqrt 2 left( {{x_1} + {x_2}} ight) + {x_1}{x_2} geqslant 2 + sqrt 2 {x_1}{x_2} + {x_1}{x_2} + sqrt 2 \ & Leftarrow {x_1} + {x_2} geqslant {x_1}{x_2} + 1 \ & Leftarrow left( {1 - {x_1}} ight)left( {1 - {x_2}} ight) leqslant 0 \ end{split}]
     
    2016年北京大学自主招生数学试题回忆版
    lanqi.org 2016年12月22
    第1页
     
    一、选择题.在每小题的四个选项中,只有一项符合题目要求.
    1.已知$dfrac{sin{x}}{sqrt{1-cos^2{x}}}-dfrac{cos{x}}{sqrt{1-sin^2{x}}}=2left(0<x<2pi ight)$,则$x$的取值范围是(  )
    A.$left(0,dfrac{pi}{2} ight)$
    B.$left(dfrac{pi}{2},pi ight)$
    C.$left(pi,dfrac{3pi}{2} ight)$
    D.前三个答案都不对
    2.$left(2+1 ight)left(2^{2}+1 ight)left(2^{3}+1 ight)cdotsleft(2^{2016}+1 ight)$的个位数字是(  )
    A.$1$
    B.$3$
    C.$5$
    D.前三个答案都不对
    3.点$P$位于$ riangle ABC$所在的平面内,使得$ riangle PAB, riangle PBC, riangle PCA$的面积相等,则满足题意的点$P$有(  )
    A.$1$个
    B.$3$个
    C.$5$个
    D.前三个答案都不对
    4.记$f(n)$为最接近$sqrt{n}$的整数,其中$nin mathbf{N}^{*}$.若$dfrac{1}{f(1)}+dfrac{1}{f(2)}+cdots+dfrac{1}{f(m)}=2016$,则正整数$m$的值为(  )
    A.$1015056$
    B.$1017072$
    C.$1019090$
    D.前三个答案都不对
    5.实数$x,y,z$满足$x+y+z=2016$,$dfrac{1}{x}+dfrac{1}{y}+dfrac{1}{z}=dfrac{1}{2016}$,则$(x-2016)(y-2016)(z-2016)=$(  )
    A.$0$
    B.$1$
    C.$-1$
    D.前三个答案都不对
    6.方程组$egin{cases}a^3-b^3-c^3=3abc,\a^2=2(b+c)end{cases}$的非负整数解有(  )
    A.$1$组
    B.$4$组
    C.$5$组
    D.前三个答案都不对
    7.$4$个半径为$1$的球两两外切,则这$4$个球的外切正四面体的棱长为(  )
    A.$2+2sqrt{2}$
    B.$2+2sqrt{3}$
    C.$2+2sqrt{6}$
    D.前三个答案都不对
    8.将$1,2,cdots,100$分成三组,使得第一组数的和为$102$的倍数,第二组数的和为$203$的倍数,第三组和为$304$的倍数.则不同的分法共有(  )
    A.$1$种
    B.$2$种
    C.$3$种
    D.前三个答案都不对
    二、填空题.
    9.已知$f(x)=3x^2-x+4$,$g(x)$为整系数多项式,$fleft(g(x) ight)=3x^4+18x^3+50x^2+69x+a,$则$g(x)$的各项系数之和为_______.
    10.$54$张扑克牌排成一列.先去掉第一张,将第二张放到最后;再去掉第三张,将第四张放到最后……以此类推,则最后剩下的那张牌是原先的第_______张.
    11.用高斯函数$[x]$表示不超过实数$x$的最大整数,则方程$nleft[2002sqrt{2001^2+1} ight]=2002left[nsqrt{2001^2+1} ight]$的正整数解有_______个.
    12.空间中的一点$P(x,y,z)$满足$exists nin mathbf N^*$,使得$|3x|^n+|8y|^n+|z|^n leqslant 1$成立,则所有满足要求的点$P$所形成的空间几何体的体积为_______.
    1.B.
    根据题意,有$sin x>0$,$cos x<0$,于是$x$是第二象限的角.
    2.C.
    因为$2^2+1=5$,且对于任意正整数$k$,都有$2^k+1$为奇数,所以 [left(2+1 ight)left(2^{2}+1 ight)left(2^{3}+1 ight)cdotsleft(2^{2016}+1 ight)equiv 5 pmod{10}.] 3.D.
    考虑到平面内使$ riangle PAB$和$ riangle PBC$的面积相等的点的轨迹为直线$BM$以及过点$B$且与$AC$平行的直线,其中$M$为边$AC$的中点,因此满足题意的点$P$有$4$个:$ riangle ABC$的重心,或者由$P,A,B,C$四点所构成的平行四边形的顶点.4.B.
    若$f(n)=k$,则$$k^2-k+1 leqslant n leqslant k^2+k,$$所以[egin{split} &f(1)=f(2)=1,\ &f(3)=f(4)=f(5)=f(6)=2,\ &cdots ,end{split} ]进而有 $$2016=dfrac{1}{f(1)}+dfrac{1}{f(2)}+cdots+dfrac{1}{f(m)}=2cdot 1+4cdot dfrac{1}{2}+6cdotdfrac{1}{3}+cdots+2016cdotdfrac{1}{1008},$$ 故$m=2+4+6+cdots+2016=1017072$.
    5.A.
    由于[egin{split} (x-m)(y-m)(z-m)&=xyz-m(xy+yz+zx)+m^2(x+y+z)-m^3,\&=mxyzleft[dfrac 1m-left(dfrac 1x+dfrac 1y+dfrac 1z ight) ight]+m^2left[(x+y+z)-m ight],end{split} ]于是所求代数式的值为$0$
    6.B.
    根据题意,有[egin{split} a^3-b^3-c^3-3abc&= a^3-(b+c)^3+3bc(b+c-a)\&=a^3-dfrac 18a^6+3bcleft(dfrac 12a^2-a ight),\&=aleft(1-dfrac 12a ight)left[a^2left(1+dfrac 12a+dfrac 14a^2 ight)-3bc ight]\&=0,end{split} ] 当$a=0$时,$(b,c)=(0,0)$;当$a=2$时,$(b,c)=(0,2),(1,1),(2,0)$.当$a e 0,2$时,有 $$a^2left(1+dfrac 12a+dfrac 14a^2 ight)-3bc>dfrac 14a^4-3bc=(b+c)^2-3bcgeqslant 0,$$于是题中方程组的非负整数解共有$4$组.
    7.C.
    棱长为$a$的正四面体的内切球半径为$dfrac{sqrt{6}}{12}a$.设$4$个半径为$1$的球的球心分别为$O_1,O_2,O_3,O_4$,则正四面体$O_1O_2O_3O_4$的棱长为$2$,故其内切球半径为$dfrac{sqrt{6}}{6}$.设这$4$个球的外切正四面体为$ABCD$,则正四面体$ABCD$的内切球半径为$1+dfrac{sqrt{6}}{6}$,故正四面体$ABCD$的棱长为$2+2sqrt{6}$.
    8.D.
    假设这样的分法存在,设三组数的和分别为$102x,203y,304z$,$x,y,zin mathbf{N}^{*}$,则$$102x+203y+304z=5050,$$ 即$$101(x+2y+3z)+(x+y+z)=101cdot 50,$$于是$$101mid x+y+z,$$ 因此$x+y+z geqslant 101$.而此时$$102x+203y+304z>102(x+y+z)>5050,$$矛盾.故不存在满足题意的分法.
    9.$8$.
    易知$g(x)$为二次多项式,设$g(x)=px^2+qx+r$,则 $$f(g(x))=3g^2(x)-g(x)+4=3p^2x^4+6pqx^3+left(3q^2+6pr-p ight)x^2+(6qr-q)x+3r^2-r+4,$$对比系数,依次解得$p=1$,$q=3$,$r=4$,$a=48$.故$g(x)$的各项系数之和为$8$.
    10.$44$.
    每一轮剩下的牌依次是[egin{split} &2,4,6,cdots ,52,54,\&4,8,12,cdots ,48,52,\&4,12,20,cdots ,44,52,\&12,28,44,\&12,44,\&44.end{split} ] 11.$4002$.
    因为$$2002cdot 2001<2002sqrt{2001^2+1}<2002cdot 2001+1,$$所以$left[2002sqrt{2001^2+1} ight]=2002cdot 2001$.于是原方程等价于$$left[nsqrt{2001^2+1} ight]=2001n,$$即$$2001n leqslant nsqrt{2001^2+1}<2001n+1,$$解得$n<sqrt{2001^2+1}+2001$,所以原方程的正整数解有$4002$组.
    12.$dfrac{1}{3}$.
    考虑第一卦限,只需要$3x,8y,zin (0,1)$即可.因此所有满足要求的点$P$所形成的空间几何体为一个长方体,体积为$$dfrac 13cdot dfrac 18cdot 1cdot 8=dfrac 13.$$
  • 相关阅读:
    自定义控件其实很简单5/12
    自定义控件其实很简单1/3
    自定义控件其实很简单1/4
    自定义控件其实很简单1/6
    ListView addfooter 没显示 footer的原因(之一)
    android 版本和 api level 的对应关系
    derby入门
    OA系统与ERP的区别
    Windows下的定时任务
    linux学习笔记——安装问题
  • 原文地址:https://www.cnblogs.com/Eufisky/p/11684713.html
Copyright © 2011-2022 走看看